• Leontiskos
    1.7k
    No, because you don't need to view the world as evil for this argument, just that preventing suffering is a priority.schopenhauer1

    The argument holds that human life is evil (or bad) on account of suffering, and therefore we should not further human life by procreation. The proposition that <Life is evil> flows from your claim about suffering, but it still seems to me obvious that this is a crucial proposition in your argument, and that it is "theological."

    So whilst I agree with what you have said there, the point is that paternalistically making a decision on behalf of someone to not prevent them from suffering, and thus basically forcing the conditions of suffering onto them, would not be respecting the dignity, as this becomes aggressive paternalism.schopenhauer1

    We force a decision upon them either way. I don't see how only one direction is paternalistic.

    In this state of affairs scenario, it is doubtful you will find this thinking absurd. That is to say, just because there isn't a particular person that this state of affairs will affect, doesn't mean we are not incumbent to prevent the situation.schopenhauer1

    The problem is that your argument is a form of metabasis, <We ought to prevent persons from suffering; preventing a person from existing will prevent their suffering; therefore we should prevent their existence>. I don't say that it is necessarily unsound, but it is not a clean syllogism. There is a quasi-equivocation on "persons."

    Our obligation is to prevent the suffering of persons, not to prevent persons for the sake of suffering. Our primary suffering-obligation is to prevent existing persons from suffering. Additional argument is required to show that this means that we should prevent persons from existing. Your aversion to suffering is overwhelming, and out of sync with common intuitions. Common intuitions merely say that suffering should be mitigated, not expunged at all costs. ...but maybe I would be better off directly addressing Benatar's argument... (and I do so at the end)

    In fact, I didn't even mention whether ONCE ALREADY EXISTING, non-existence is or is not considered a harm. You cannot put the genie back.schopenhauer1

    Why not? Does existence suddenly become a non-harm once someone is born? Is life bad before we are born and good after we are born? So that we must avoid it before we are born and embrace it after we are born?

    Let's move to the consensual variant, which seems to me more fruitful:

    Well, now you've changed it. If he asked, and everyone consented, ethically speaking, this isn't violating an ethic. Whether this is the right "solution", I don't know, because I don't believe already-existing to be symmetrical for never-existing.schopenhauer1

    Again, why are they not symmetrical? I am guessing that unhappy people correlate to antinatalism and happy people correlate to "pronatalism," because there is symmetricity. Again, the whole thing is based on the question of whether life is good or bad, and that determination should hold steady. So if you really think life is bad then you should think that other people should think that life is bad, and that other people should consent to painless euthanization. If euthanization is not the rational choice for living persons, then why would you promote antinatalism? (Note that when I talk about the "rational position," what I mean is that this is the choice that the rational person ought to freely choose for themselves.")

    But once someone has X done upon them, if it means that they have abc experiences, and they value them, I see no need to get rid of them, unless indeed they thought they were were worthless.schopenhauer1

    If X doesn't want to be euthanized because they find life beautiful and valuable, then either they are irrational or else the antinatalism thesis suffers a blow. That person would literally resent your antinatalism, because it sought to "paternalistically" prevent their fulfilling life. These two realities are directly opposed. It is not faulty logic.

    Rather, we are NOT LIVING for that value, but rather, preventing that negative state of affairs from befalling someone.schopenhauer1

    It seems to me like a rationalization of pusillanimity: fear of life for fear of suffering. The antinatalist would apparently counsel the unborn to opt out of life for fear of suffering.

    Do you agree that, one way or another, we must make a choice for the unborn? That to give birth is to choose life for them, and that to abstain from procreation is to choose non-existence for them? (Really "them," as I am now swimming in the metabasis). I don't accept the purported neutrality of the antinatalist position, as if so-called "paternalism" is not inevitable.

    My point was that empirically-speaking, in the real world, there are no such charmed lives, so it is de facto out of the question other than a thought experiment. Supposing only a pin-prick was the suffering, I guess the scenario could be reconsidered.schopenhauer1

    Reconsidered on what basis? I am offering a reductio, and if your argument succumbs then the argument itself is problematic, as it "proves too much."

    So for example.. What if when you stab someone, they reanimate every time you do it instantly.. would that be wrong?schopenhauer1

    It wouldn't be wrong in the same way as it is now. But your theoretical does not function as a reductio to any argument that I have offered, and that is the primary difference.

    Benatar thinks indeed, being that no one being deprived of this "almost charmed life", there is no foul. No person harmed, no foul. Rather, the violation still takes place in this scenario.schopenhauer1

    The problem occurs if this is a valid argument:

    1. Suppose every living human being is guaranteed a pinprick of pain followed by 80 years of pure happiness.
    2. [Insert Benatar's antinatalist argument here]
    3. Therefore, we should never procreate

    Are you starting to see the reductio? The reductio has force because we know that any (2) that can get you from (1) to (3) is faulty argumentation.
  • Fire Ologist
    381
    I wouldn’t argue with you that I’ve made arguments based on emotion in here. But there is a basic premise that suffering qua suffering is only bad, so inflicting suffering on another is only bad, and inflicting suffering on another is unethical. These notions arise out of emotion, and raw experience (unless there is some heretofore 11th commandment that the antinatalists dug up.). Suffering itself involves emotions, physical states and psychological reactions to those states, so bringing emotion into it isn’t a non-sequitor.

    But, true, I’m not just building analytic proofs here.

    1. There is no ethical way to treat non-existing people,Fire Ologist

    The ethics are to do with our actions now. Not unborn people.AmadeusD

    Then it would stand to reason you are an anti-abortionist? Someone who would not look twice pulling out in the road? Wouldn't remove broken glass from a playground? These are all potential harms to no one in particularAmadeusD

    Schop said, and I agree, that ethics only exist where people and exist, and in particular, where more than one person exists. The ethical rule at issue is: it is always wrong to inflict suffering on others and/or to do so without consent. And the ethical solution is to stop making people.

    While I disagree with the rule (and I don’t thereby think inflicting suffering is good, just that it is not universally always wrong to inflict any suffering without consent), leaving the rule as is anyway, there is a distinction between an ethical rule that prevents harm to people you don’t know in particular, and a rule that prevents harm in people people that don’t exist.

    It is true that, if all suffering infliction is wrong, the unethical behavior is “to do with our actions now” and the unethical person is the one who exists, inflicting the suffering. But in all of the above scenarios, in your quote, there are already existing victims of the harm. The antinatalist isn’t looking out for the fetus who will be a person; the antinatalist is urging no one gets pregnant. There is no person existing that is the one towards whom the ethical act is directed. There is only the antinatalist claiming ethical treatment of the beneficiaries of that ethical treatment, namely beings (I guess you can call them human beings) that never come to be. There is not only no particular subject of the antinatalists actions, there is no potential ethical subject.

    Too leave the glass on the playground is potentially inflicting harm (not necessarily, whereas necessary suffering is part of the antinatalist solution) on anyone who plays there. The people who might or might not play there actually exist while I consider whether or not to remove the glass. The non-procreated are not beings at all. So if two ethical beings must exist for there to be an ethics between them, those beings exist in the playground scenario but do not exist in the antinatalist world. We can’t prevent suffering in someone until someone exists, at least any reason we would have to prevent further existence is not a matter of ethics towards that further existing being, because there is no one or thing there to be ethical towards.

    If I use the playground analogy, the antinatalist reasoning to me would lead to solutions like “there should be no playgrounds” or “because children at play can cut themselves on glass, there should be no children.”

    Truth is antinatalism solves every math problem, every philosophical conundrum, every imperfection - all of gone to history. Problem is, people like playgrounds, despite the glass. All ethics and logic and suffering be damned when there is good sliding board around.
  • ENOAH
    664
    I am making a rule that says I should not be making rules.Fire Ologist

    Totally

    Antinatalism isn’t tailored to the specific problem it is trying to prevent, and is way overboard of a response to just suffering.Fire Ologist

    And, if you don't mind, add: and, a response directed at the wrong party. If you want to end suffering, end mind's constructions, and attachments thereto. Why end a species?
  • schopenhauer1
    10.3k
    The ethics are to do with our actions now. Not unborn people. The potential suffering itself is not hte moral crux. The action that (on the balance of probabilities) will make it come about, are. This is a gross oversimplication (or, overcomplexification, depending where you stand) of the point of antinatalism.AmadeusD

    Yep

    Then it would stand to reason you are an anti-abortionist? Someone who would not look twice pulling out in the road? Wouldn't remove broken glass from a playground? These are all potential harms to no one in particular (the A-symmetry argument beats this anyway). A starker example is, why keep NICU's sterile? Hehehe.AmadeusD

    Yep/.. I am not sure why @Fire Ologist isn't quite getting this.

    Benatar, particularly, addresses this issue. It is far more likely the figure is closer to 80% (this is interpolation based on my thoughts along with his arguments around it). Polly-Anna syndrome is rife. Most people are genuinely mistaken about how often they suffer. That said, I'm unsure this is a particularly strong anti-natalist argument anyway. I don't care what living people think about their lives. The vast majority of anti-natalists hold that the living have a deep interest in continuing to live. Perhaps there are situations in whcih this isn't the case, but overall, its hard to find examples of that.

    Your earlier two objections are to stronger arguments, and I think your objections are just your taste. They aren't logic objections or reasonable ways around the claims made. They just illustrate that you do not accept htem, prima facie. That's fine. None of that has to do with the strength or weakness of hte anti-natalist position other than how it strikes you (weakly, it seems).
    AmadeusD

    Yep.

    All of these are basically my points as well.
  • schopenhauer1
    10.3k
    Antinatalism isn’t tailored to the specific problem it is trying to prevent, and is way overboard of a response to just suffering.Fire Ologist

    Again, you ignore what it means to cause suffering and back-filling the justification after-the-fact with palliative ethics. You don't intentionally cause the situation for which you think you then have the remedy. And since you really ignored my argument, and simply show your distaste, I don't know what else to say other than you have failed to show any meaningful objections other than indeed your distaste for the logic.
  • schopenhauer1
    10.3k
    When thinking ahead for the unborn-yet-to-be-procreated persons, the potential ones antinatalism is trying be ethical toward, couldn’t we just as easily instead think of those unborn persons and make the rule “one cannot deprive someone of happiness without their consent.”Fire Ologist

    No, causing happiness is not an obligation in the way NOT causing harm is. Causing pain in order for them to also have happiness is still a violation of non-malfeasance.

    Happiness-promoting is not obligatory, and especially so if to promote happiness you knowingly create the conditions for someone else to be harmed. Procreation creates the very need for palliative measures to mitigate suffering. Preventing harm by not procreating avoids creating this need altogether.
  • Fire Ologist
    381



    and, a response directed at the wrong party. If you want to end suffering, end mind's constructions, and attachments thereto. Why end a species?ENOAH

    See, that reads to me like a breath of new air in the conversation. Sort of procreates some breathing space.

    Reminds me that we are talking about making a rule that must guide my actions. It’s not just an analytic exercise with suffering and ethical rules as its parts. The argument for antinatalism is also about my response, like where you said it was a response directed at the wrong people.

    After we’re done with the logical analysis, antinatalism is a call to action. It says if we work hard enough together, maybe in a few generations, we will finally all do the ethical thing and choose to be antinatalists, then, the human species will be gone, and with it, gone is all suffering, and those last people can say “we, now the most ethical generation, the ones who have inflicted the least suffering on others, we bid to those who discovered antinatalism our humble gratitude for showing us the ethic, and bid everyone else, including ourselves, I guess, good riddance.”

    While I think we mean different things when you say “end mind’s constructions and attachments thereto”, I would use those same words to point to the fact that all ethics is in our heads. Ethics is about what we physically do when we interact with other people in the world, but the ethical parts of those actions exist only in my head, in our heads. We must construct all of this, and together for it to be ethics. I would also instead use some of your words to say, “while we are always attaching and detaching from constructions, to be ethical is to take care of what you attach to and detach from.”

    But we need both the analytic approach, and this raw, more contextual (aware of aware-ing) view to find an ethical norm.

    My attempt at the analytic syllogism:

    Life is suffering.
    No one should intentionally cause suffering.
    Procreating is intentional infliction of suffering…
    Therefore, antinatalism.

    Negatively put, we should not inflict suffering, so we should not procreate. Or more positively put, it is right not to procreate, or else you would intentionally cause suffering.


    We’ve constructed out of suffering this new ethic. (It had to be new because people had to be here first to construct it, it is newer than us at least.)

    Now of course antimatalism is more that. And granting the premises, it’s a sound argument. That may be enough for many intuitions.

    But what can we know and say about this syllogism?

    P1: Life is suffering.

    Maybe. Maybe because of life, we have something to compare suffering with in order to identify that life includes suffering. Without something good in life to compare to, how would we recognize suffering. We had to have a happy finger first before we could say that on prick was suffering. But then, does that mean the good and happy finger is a cause of the suffering too?

    But maybe life is living. And living is many things, with the many things we live with. One of them is suffering. One of them is ecstasy. One of them is sleeping. One of them is a pin-prick, or reading. Life is reading, right now.

    Also, this premise is where we assume a sub premise “suffering is only bad.” Suffering is bad, but it is not only bad. Some suffering is called work. Some called really hard work. Some called loving. Some called longing. Some longing is suffering deeply. Some longing is not.

    I think many are willing to say there is enough suffering assured in life that it’s not worth breaking it down into how much or how little there is that it would start to change the calculus of the syllogism. Many would argue life pivots from suffering too much to suffering enough, and therefore “Life is suffering” is a valid premise, period, end of discussion.

    So we’ll move on, under protest.

    P2: No one should intentionally cause suffering.

    Sounds like a nice sentiment from the start. But it depends on your view of what suffering is from P1 if you would make this an absolute. If all suffering is bad, then yeah, absolutely no one should inflict it on anyone else. But many of us have had suffering inflicted upon us without our consent, at great cost, causing deep suffering, only to later count the experience as a a good one. That’s the constant life of a child. Some suffering can lead to tremendous things that would not have been what they are without the suffering. So if not all suffering is bad, what is wrong with causing it in another?

    So this premise only works if all suffering is bad. So my protest from P1 is rearing its head.

    That means to me I should try to tailor P2 to keep the argument flowing.

    How about new P2: “No one should intentionally harm or injure another for the sole purpose of causing them to suffer.”

    Sounds stronger, and accounts for the good of inflicting valuable suffering, but this will be a problem when it comes time to choose whether to procreate. Now with new premise 2 we have to have a baby already born who we can physically “harm and injure” before we might unethically do so for “the sole purpose of causing them to suffer.”

    I just disagree that intentionally causing suffering, the heart of this ethical rule, is necessarily something that should never be done, it’s not clearly a universal. It’s not self-evident.

    It needs work, as does my argument against it, but let’s press on.

    P3: Procreating is intentional infliction of suffering.

    This conflates begetting, or giving new life, with infliction on a subject. When we inflict, we inflict upon. There must be an object that we inflict something specific like suffering upon. But that object is missing in the syllogism. Take the antinatalist negative approach and flip it, and you see the hole, the missing object when one tries to inflict something onto the unborn, the non-existent.

    If we do not procreate we will not inflict suffering upon……..who? (I get it, the answer is: on the possible child that would have been had you gotten pregnant.). But really, who benefits when I do not have a kid so I can not inflict suffering? Can I say I benefited 10 babies because I was going to have at least 10 kids, but now since I’m an antinatalist, I benefited 10 people? That seems really odd. But if making up some number of never-existing people as beneficiaries of my good antinatalist deeds is odd at all, so is saying I benefitted one person. There is no one person who benefits if you conflate the Infliction of suffering with procreation and respond by not procreating. I’m just procreating. There is no one there to enjoy my mon-procreation with, as there is no one to say “thanks for not inflicting suffering on me” because I didn’t create any such life.

    Secondly, there is a bias in the word “inflict”. We don’t inflict levity and happiness. We inflict pain and suffering. To procreate need not be an “infliction” of anything at all. Maybe it is another place to reword the premise better.

    But thirdly, premise three is kind of premise 1 reworded. To say procreation is inflicting suffering is like saying life is suffering. Life, the tug and pull of being and becoming, or just the becoming of being, or just the “ing” of suffering. If life is suffering procreating is building new suffering or inflicting suffering.

    This just goes back to the arguments against premise 1, that life can’t only be suffering for suffering to be distinguishable as a feature of life at all - life is more than suffering, and more than enough good to enable appropriate attachment to other features of life besides suffering. And suffering isn’t so bad that you must never cause it in another.

    But, Conclusion: Antinatalism.

    Logically flows from the premises as they are meant by the antinatalist. Suffering is truly excruciating and life is suffering, so since procreating makes new lives of suffering, we should not procreate. Certinly uses logic.

    But only compelling if you think suffering is so bad, that suffering is the definer of becoming a human being, that we can’t see any good in suffering, that we can’t see other things besides suffering more valuable to us, that we can’t use our suffering to construct other things, and that if we inflict suffering it is never for good, it is always an unethical act. I just disagree with all of those observations. Together they mean an end to procreation is wildly inappropriate a response. It’s an emotional response to suffering, not something clear enough to construct an ethical norm.

    It’s a slap in the face of the Dionysian. Mother Nature inflicted humanity on the universe, why would we judge her so harshly and end her creation, for the sake of each other not having each other?

    I don’t see wisdom in turning against life itself because of suffering.

    We should turn against the suffering, not the life that begets it or not even the ones who inflict it.

    The law should be, because life involves suffering, we should give relief and kinship, so that others can know life with less suffering, but others can know life.

    My arguments don’t seem compelling enough yet either. But that just means I need to keep searching for the words, suffering through the birth of better words, constructing something to attach to that accounts for more than the last attachment.
  • schopenhauer1
    10.3k
    Without something good in life to compare to, how would we recognize suffering. We had to have a happy finger first before we could say that on prick was suffering. But then, does that mean the good and happy finger is a cause of the suffering too?Fire Ologist

    Suffering is intrinsically negative and harmful regardless of comparative happiness. The existence of suffering is ethically significant on its own, as it involves harm and distress that should be prevented when possible.

    But maybe life is living. And living is many things, with the many things we live with. One of them is suffering. One of them is ecstasy. One of them is sleeping. One of them is a pin-prick, or reading. Life is reading, right now.Fire Ologist

    Promoting happiness at the expense of causing harm violates the principle of non-malfeasance by justifying harmful actions for potential positive outcomes. It involves using individuals as means to an end, imposing projects on them without their consent, and treating them as tools for achieving broader goals. Ethical consistency requires prioritizing the avoidance of harm and respecting individuals' autonomy and dignity, rather than foisting known and unknown conditions of harm upon them, for the sake of some positive-ethical external objectives.

    Also, this premise is where we assume a sub premise “suffering is only bad.” Suffering is bad, but it is not only bad. Some suffering is called work. Some called really hard work. Some called loving. Some called longing. Some longing is suffering deeply. Some longing is not.Fire Ologist

    While some people argue that suffering isn't all bad and can lead to growth, this idea falls short when you look at the bigger picture. First off, suffering is inherently harmful. Even if it might lead to some positive outcomes, the immediate distress and pain it causes shouldn't be ignored or justified. The principle of "do no harm" should always come first, and causing suffering to potentially promote happiness just doesn't cut it ethically. Plus, making someone else suffer because you believe it's good for them totally disregards their right to make their own choices. It's like deciding for them that the end justifies the means, which is a huge overstep.

    Treating people as tools for achieving a goal (like personal growth through suffering) is another major issue. People aren’t just instruments for projects we believe in. Everyone has different ways of dealing with and understanding suffering. What might be a meaningful struggle for one person could be devastating for another. This unpredictability means you can't assume suffering will lead to positive outcomes for everyone.

    Respecting personal autonomy is crucial. Everyone should have the right to decide whether they want to endure suffering for a potential benefit. Foisting this decision on someone else is ethically wrong and incredibly paternalistic. It’s like saying, "I know what's best for you," which often isn't the case. Growth and fulfillment can happen through positive experiences and challenges that don't involve suffering.

    Finally, there's a big difference between preventing harm (preventative ethics) and trying to fix it after the fact (palliative ethics). Creating situations where suffering occurs and then trying to justify it with potential benefits is backwards. It’s better to avoid causing harm in the first place. All in all, assuming that suffering is valuable and imposing it on others without their consent is unethical and dismissive of individual autonomy and well-being.

    After reviewing your P2, that last response answers that as well, so moving on...

    This conflates begetting, or giving new life, with infliction on a subject. When we inflict, we inflict upon. There must be an object that we inflict something specific like suffering upon. But that object is missing in the syllogism. Take the antinatalist negative approach and flip it, and you see the hole, the missing object when one tries to inflict something onto the unborn, the non-existent.

    If we do not procreate we will not inflict suffering upon……..who? (I get it, the answer is: on the possible child that would have been had you gotten pregnant.). But really, who benefits when I do not have a kid so I can not inflict suffering? Can I say I benefited 10 babies because I was going to have at least 10 kids, but now since I’m an antinatalist, I benefited 10 people? That seems really odd. But if making up some number of never-existing people as beneficiaries of my good antinatalist deeds is odd at all, so is saying I benefitted one person. There is no one person who benefits if you conflate the Infliction of suffering with procreation and respond by not procreating. I’m just procreating. There is no one there to enjoy my mon-procreation with, as there is no one to say “thanks for not inflicting suffering on me” because I didn’t create any such life.
    Fire Ologist

    This principle doesn't need to be about a specific person who benefits or says thanks. It's about the ethical stance of the decision-maker to prevent harm. The decision not to have children because you want to avoid causing potential suffering is about maintaining an ethical standard. Saying "who benefits?" misses the point because it’s not about imaginary future people celebrating your decision. It's about your commitment to an ethical choice that avoids creating a scenario where suffering is possible. It’s not about benefiting non-existent beings; it’s about making a responsible and ethical decision based on the understanding that creating life inevitably involves creating suffering, and the ethical imperative to avoid this suffering outweighs other considerations, and the recognition that not doing so, is a paternalistic overstep in foisting one's projects onto another, using them. So, this strawman argument oversimplifies and misrepresents (this particular) antinatalist position.
  • ENOAH
    664
    But maybe life is living. And living is many things,Fire Ologist

    Is this not the long and the short of it?

    With a Bricolage of logic and reasoning, a skilled artist can construct complex hypotheses to justify many things in ethics. As much as we like to think ethics transcends us, that thought too, emerged because it is functional to view ethics that way. Contra Descartes et. al., the only thing this organism can claim with certainty is that this organism is. Period. The same cannot be said of any of its constructions from "I" to "all life is Dukkha," to "thou shalt not x".

    So does it not boil down to: life is living. We humans make the distinction suffering/no suffering. These, and like distinctions are how our world turns. But with antinatalism we are clearly going too far. People think asceticism a radical approach; how much more is extinction?

    Even practically. If you can mobilize an entire species to eliminate one of the three strongest drives--to perpetuate living, and to bond with one's offspring--you might as well exhaust first, all other efforts to change the conditions. If the livestock are suffering under our current conditions, the solution is to change the conditions, not to sterilize the cattle.

    As much as I have been turned on to Schopenhauer. This is the corrupted truth which emerges out of the insistence that the will is both the seat off the suffering and our essential human nature. The will constructs suffering. Our human nature may experience pain, the absence of pleasure, but it does not construct anything out of that; it responds to it, just as it responds to pleasure.

    The so called will is the name we give to those dynamics of Mind. Difficult to escape, but actually malleable.
  • schopenhauer1
    10.3k
    We force a decision upon them either way. I don't see how only one direction is paternalistic.Leontiskos

    Because only when born is it paternalistic, as there is someone aggressed. The aggression only works one way.

    Our obligation is to prevent the suffering of persons, not to prevent persons for the sake of suffering. Our primary suffering-obligation is to prevent existing persons from suffering. Additional argument is required to show that this means that we should prevent persons from existing. Your aversion to suffering is overwhelming, and out of sync with common intuitions. Common intuitions merely say that suffering should be mitigated, not expunged at all costs. ...but maybe I would be better off directly addressing Benatar's argument... (and I do so at the end)Leontiskos

    No, going back to the Kant thread, not preventing all counts of suffering BECAUSE (X reason that parent has), IS using as means to an ends, overlooking dignity. It is this crux of dignity that you are missing. It was unnecessary, other than promoting some cause.. That project is done on behalf of the child- hence aggressive paternalism.

    Why not? Does existence suddenly become a non-harm once someone is born? Is life bad before we are born and good after we are born? So that we must avoid it before we are born and embrace it after we are born?Leontiskos

    No, because again, one could have been prevented, one violated and justified with mitigation or because of X conflicting reason of the parent, to be carried out for the child, but can never be the child.. there was no child to begin with to even need happiness.. nor to need the suffering which ultimately comes along with it... And again, "merely" becomes a cover for everything. I merely let that thing hurt you because in the future, it could be in your interest. There is an reductio for ya.

    Again, why are they not symmetrical? I am guessing that unhappy people correlate to antinatalism and happy people correlate to "pronatalism," because there is symmetricity. Again, the whole thing is based on the question of whether life is good or bad, and that determination should hold steady. So if you really think life is bad then you should think that other people should think that life is bad, and that other people should consent to painless euthanization. If euthanization is not the rational choice for living persons, then why would you promote antinatalism? (Note that when I talk about the "rational position," what I mean is that this is the choice that the rational person ought to freely choose for themselves.")Leontiskos

    I've already answered and you conveniently ignored and cherry picked so I will present it again:
    Can it be that once existing, different priorities are considered in regards to harms and goods? Perhaps. For example, I wouldn't recommend forcing X upon someone. But once someone has X done upon them, if it means that they have abc experiences, and they value them, I see no need to get rid of them, unless indeed they thought they were were worthless. So perhaps nothing should have been done to that person, but once it's done, it doesn't take away the value they might have gotten. They do not have to be mutually exclusive. This is a trap many anti-AN arguments fall under. If there is good from a bad, then the bad must not be that bad. That is faulty logic.schopenhauer1

    If X doesn't want to be euthanized because they find life beautiful and valuable, then either they are irrational or else the antinatalism thesis suffers a blow. That person would literally resent your antinatalism, because it sought to "paternalistically" prevent their fulfilling life. These two realities are directly opposed. It is not faulty logic.Leontiskos

    Actually, if we want to use Benatar here, that matters not. If no ONE is deprived of the good, then the decision does no good or bad that "no benefits" took place for someone. The onus is on the creating the bad "for someone". I might even argue that creating bad "for no one" is not good or bad either. It can only be formulated as a maxim for someone making the decision.

    And yes, the ethic assumes that creating suffering is more important ethically than promoting happiness. And hence why even the pinprick argument is a sort of shrug because due to the non-existing people to not care about it, it becomes moot. It's only YOU the person existing crying on the sidelines over non-existent spilt milk.

    Do you agree that, one way or another, we must make a choice for the unborn? That to give birth is to choose life for them, and that to abstain from procreation is to choose non-existence for them? (Really "them," as I am now swimming in the metabasis). I don't accept the purported neutrality of the antinatalist position, as if so-called "paternalism" is not inevitable.Leontiskos

    No it's precisely this kind of formulation I am against. The only scenario where something would happen for/to them is the one where a person results, none of the others. At this point, it might be helpful to draw a table...


    Reconsidered on what basis? I am offering a reductio, and if your argument succumbs then the argument itself is problematic, as it "proves too much."Leontiskos

    No, not really. When you create a fantasy world and that changes the very terms of how existence works, I don't see that as proving anything. What if gravity didn't exist? How would that change ethics? What if time and space could be changed so that we can redo actions? Again, none of this is this world. We can argue facts, but then at least we are arguing what is the case, and not hypotheticals that change how ethics would work because circumstances of the very conditions for ethics have changed.

    It wouldn't be wrong in the same way as it is now. But your theoretical does not function as a reductio to any argument that I have offered, and that is the primary difference.Leontiskos

    It's to show an illustration of what is going on when you change things to a certain extent.
  • Leontiskos
    1.7k
    No, not really. When you create a fantasy world and that changes the very terms of how existence works, I don't see that as proving anything. What if gravity didn't exist? How would that change ethics? What if time and space could be changed so that we can redo actions? Again, none of this is this world. We can argue facts, but then at least we are arguing what is the case, and not hypotheticals that change how ethics would work because circumstances of the very conditions for ethics have changed.schopenhauer1

    Er, I think antinatalism is dead in the water due to this argument:

    • Procreation is permissible in an all but perfect world
    • Benatar's argument excludes procreation even in an all but perfect world
    • Therefore, Benatar's argument is unsound

    What is your counter supposed to be? "No, because counterfactual analyses aren't allowed"? I would suggest reading about the fallacy of "proving too much." Benatar's argument cannot account for the fact that procreation is obviously permissible in an all but perfect world. Benatar would not allow procreation before removing that pinprick. That's crazy. Benatar is irrationally opposed to life, and he would be irrationally opposed to life even in the best of circumstances. Indeed, his irrational argument opposes life even in the best of circumstances!
  • Leontiskos
    1.7k
    I think antinatalism is inherently bound up with Gnosticism. This is because it opposes the natural order, and to oppose the natural order requires appealing to some vantage point outside of the natural order. “You shouldn't procreate because the world is evil, addled by suffering.” But how do we know that the world or nature is evil? Surely nature did not tell us such a thing, nor cognitive faculties formed by nature. So then how would we know that it is evil? As the Gnostic says, it must be knowledge received from some god who is opposed to the god of this world (and the nature of the world it created). So again, antinatalism is theological in the sense that it presupposes nature-transcending knowledge.

    For example, given that Benatar’s argument opposes the natural order, it cannot have been derived from the natural order. So if Benatar really thinks his argument holds good, then he must hold that his own mind and the knowledge it has come to know is super-natural, transcending nature. If we are limited to nature then we cannot contradict nature. Where does such a mind or such knowledge come from, if not from nature? Either the Gnostics are correct and it comes from the true god, or Christians are correct and it comes from demons. Or else it is just hopelessly mistaken and a product of merely human irrationality.
  • AmadeusD
    2k
    P1: Life is suffering.Fire Ologist

    This is definitely the most arguable aspect of the whole thing. I actually do sometimes swing on this one, but as a living person it hits me as patent that I'm off the rails at that point. I am not important to my own considerations here. It is whether or not causing people to exist is ethical. If you are (as I would put it) deluded into thinking your life is, on balance, purposeful and happy, you will reject this premise and (whether erroneously or not) be fine with procreation. The other premises don't seem to be shakeable in this way.

    Suffering itself involves emotions, physical states and psychological reactions to those states, so bringing emotion into it isn’t a non-sequitor.Fire Ologist

    This is true, but where we are dealing with non-existent people this is not relevant other than to assign a position to oneself as a result of their emotional reaction to whatever proposition..

    But in all of the above scenarios, in your quote, there are already existing victims of the harm.Fire Ologist

    There are clearly not. There are potential victims. This is why the analogy holds, for the most part. And taking this straight to your conclusion of "no playgrounds", yes, that's right, but antinatalists don't confuse the issue:
    No humans. Not not playgrounds. Let the people who exist use hte playground, for reasons your point out that would make the "no playgrounds" conclusion stupid as heck. That said, it seems fairly clear that's not hte intention. THe intention is to leave the playground (world)as is, and remove the potential sufferer as it is (on this account) an unavoidable consequence of being one. We're not trying to get rid of oceans to avoid drownings - we're trying to stop people swimming where drowning is a clearly likely consequence (very shaky analogy, but there you go - can refine later if needed).

    Is this not the long and the short of it?ENOAH

    Yes. But "many things" includes many things one would not trade non-existence in for, if the choice were at all a coherent one. If someone is "many things" but in there is psychopathic sexual deviant, we aren't inclined to let it roll.

    Er, I think antinatalism is dead in the water due to this argument:Leontiskos

    That isn't an argument.

    This is because it opposes the natural order, and to oppose the natural order requires appealing to some vantage point outside of the natural order.Leontiskos
    For example, given that Benatar’s argument opposes the natural order, it cannot have been derived from the natural order. So if Benatar really thinks his argument holds good, then he must hold that his own mind and the knowledge it has come to know is super-natural, transcending nature.Leontiskos

    Simply put, what? Total non sequitur. Sorry to say, but this just hits as red herring designed to impugn the logic of an argument on a naturalistic basis. Who gives a flying toss if its "against the natural order"? You'd need to establish what that is, and how it is derived (in light of hte extremely novel consciousness humans currently enjoy (see what i did there..)) in a way which could logically make Benatar's points untenable. You can't support these claims - only make them. I do smell the overwhelming aroma of religiosity coming through here... where did the evil bit come in?
    Either, you have that knowledge, or his arguments are as good as yours. One for thee, one for me. Though, if you've simply been overzealous and mean only to posit that his arguments are inadequate and not that his position is wrong due to other arguments, sure. It reads as if you're trying to present a logical basis for rejecting his arguments. But, they literally boil down to your tastes regarding epistemology it seems. And I'd say your comments there are.. bizarre... That said, you'd very much enjoy prof. Tim Mulgan's Ananthropocentric Purposivism.
  • schopenhauer1
    10.3k
    I think antinatalism is inherently bound up with Gnosticism. This is because it opposes the natural order, and to oppose the natural order requires appealing to some vantage point outside of the natural order. “You shouldn't procreate because the world is evil, addled by suffering.” But how do we know that the world or nature is evil? Surely nature did not tell us such a thing, nor cognitive faculties formed by nature. So then how would we know that it is evil? As the Gnostic says, it must be knowledge received from some god who is opposed to the god of this world (and the nature of the world it created). So again, antinatalism is theological in the sense that it presupposes nature-transcending knowledge.Leontiskos

    Antinatalism need not be synonymous with philosophical pessimism, though the two concepts are often related. Antinatalism only requires certain logical principles to be true. If we believe in a negative ethics where it's best not to cause suffering, harm, or use others, it is imperative not to overlook the consequences of procreation, even if motivated by a positive ethical cause, like promoting happiness.

    Everything follows from this principle. There are no non-existent people who benefit from being born; rather, it is about preventing the suffering, harm, and violation of existing people, if they are born.

    Gnostic-like elements can be used to justify why suffering is as bad or worse than people initially think. While these more theoretical notions of suffering can be indulged, antinatalism does not hinge on them. If we do take these aspects seriously, there is an argument, that it appears that once born, we are trapped in a cycle of unfulfilled desires with little chance for genuine repose. We can only mitigate this through psychological and physical means, such as fulfilling needs and wants or using mental exercises to manage angst, but this will never satiate, as Schopenhauer noted in detail.

    Beyond the inherent suffering of existence, life imposes contingent suffering based on situations and circumstances beyond one's control. These include diseases, natural disasters, social injustices, and personal losses. The world subjects individuals to random and often severe hardships, from debilitating illnesses to the trauma of losing loved ones, and the cruelty of societal inequalities. Even small daily annoyances—everything from traffic jams, minor illnesses, interpersonal conflicts—accumulate over time, adding to white noise baseline of human suffering. By choosing to procreate, we are subjecting new individuals to these unpredictable and uncontrollable harms. This contingent suffering exacerbates the moral issue of procreation, as it means bringing someone into a world where their well-being can be drastically compromised by forces they cannot influence or evade.

    While there are inherent goods in life, claiming that these justify causing harm oversteps a deontological point. The inherent goods can be summarized as follows:

    -Physical pleasure: The simple joys of sensory experiences.
    -Human or animal connections: Friendships, romance, group activities, bonding, nurturing, etc.
    -Flow states: Getting "caught up" in something challenging and engaging.
    -Aesthetic pleasures: Enjoying art, humor, nature, music, literature, etc.
    -Learning: Gaining new concepts or skills, engaging in hobbies.
    -Achievement: Accomplishing goals and feeling a sense of accomplishment.

    These goods add value to life. But to assert that these goods justify the creation of life, with its inevitable suffering and harm, oversteps a fundamental deontological principle. This principle maintains that we should not unnecessarily cause harm for people (i.e don’t create circumstances of harm for someone that did not need existing harm reduced with a lesser harm)- this is regardless of potential positive outcomes. Procreation involves deciding that potential pleasure and meaning for new beings outweigh the certain suffering they will encounter, which is a significant and hubristic ethical assumption.

    Humans are unique in their self-reflection. Unlike other animals that live in the moment, generally absent existential or moral considerations, humans do and can make self-consciously deliberate choices. Other animals exist within the simplicity of their instincts, unburdened by the weight of self-awareness. In contrast, humans have been metaphorically kicked out of paradise because we possess the knowledge of good and evil, the capacity for existential contemplation.

    Choosing to bring someone into existence means deciding that they should live out the challenges and issues we deem meaningful. This is a profound choice, not a natural inevitability. People seek pleasure and meaning in various ways, but deciding for others what is meaningful or subjecting them to a lifetime of issues is something that is paternalistically assumed to be imposed for another.

    You claim that the antinatalist is secretly religious, but even the most atheistic person promoting having kids is really the pseudo-religious because they see life as something necessary for someone to live out. Indeed the project of life becomes more important than the harms that that person would experience. It is this overlooking of suffering for the project of life that I am concerned with. It is this idea that is the crux of the dignity violation. It’s no “mere” here, it’s full stop.
  • schopenhauer1
    10.3k
    There are clearly not. There are potential victims. This is why the analogy holds, for the most part. And taking this straight to your conclusion of "no playgrounds", yes, that's right, but antinatalists don't confuse the issue:
    No humans. Not not playgrounds. Let the people who exist use hte playground, for reasons your point out that would make the "no playgrounds" conclusion stupid as heck. That said, it seems fairly clear that's not hte intention. THe intention is to leave the playground (world)as is, and remove the potential sufferer as it is (on this account) an unavoidable consequence of being one. We're not trying to get rid of oceans to avoid drownings - we're trying to stop people swimming where drowning is a clearly likely consequence (very shaky analogy, but there you go - can refine later if needed).
    AmadeusD

    Yep :up:
  • Fire Ologist
    381


    P1: Life is suffering.
    — Fire Ologist

    This is definitely the most arguable aspect of the whole thing.
    AmadeusD

    I agree. But the rest isn’t fairly arguable?

    It is whether or not causing people to exist is ethical.AmadeusD

    Yes.

    But there’s no reasonable quarrel with “procreating is inflicting, and inflicting suffering because it inflicts life which is suffering?” Seems subject to scrutiny, and potentially analogous to “feeding is inflicting suffering, and opening a window to let in some fresh air is inflicting suffering”, because all of these promote life, like procreation promotes life which is suffering. I might not only have to be an antinatalist, I might have to be an anti-hydrationist, because giving a thirsty person a glass of water, is like giving birth to a new person.

    And no need to consider what other things we cause by not procreating? As long as we don’t inflict suffering we will be doing good in this world, be good for this world - not arguable?

    If you are (as I would put it) deluded into thinking your life is, on balance, purposeful and happy, you will reject this premiseAmadeusD

    Getting a little emotive here, which you criticized me for above. You mean, one person who would reject this would be someone satisfied with experiences of purpose and happiness instead of suffering. You can call this one person deluded. Maybe they are. But then who is anyone to judge someone else’s self delusions and their effect on the balance of their own suffering? Must we stop the delusional thoughts of others with our own better thoughts? Like the wonderful thought of antinatalism?

    And why are happiness and/or purpose, as you frame the delusion, the only counters to suffering? If you are (as I would put it) deluded into thinking life is, on balance, suffering, then you would reject anyone who viewed any life as on balance, not suffering. Screw purpose. I’m enjoying just trying to argue with you here.

    Maybe you are right that the suffering in life is the most arguable premise, but the other premises utterly rely on suffering and can be shown almost as anrguable even granting that life is suffering.

    We're not trying to get rid of oceans to avoid drownings - we're trying to stop people swimmingAmadeusD

    Antinatalism analogized to, ironically, a life guard, keeping people out of the dangerous waters. That’s backwards. Antinatalism would eliminate the lives to guard, not merely keep lives on the land to live safely. A lifeguard would inflict a riddance of the ocean to those safely on land, not a riddance of living, like antinatalism would.

    Living is simply different than suffering and cannot be summarized as only suffering.

    Bottom line to me, in a raw, physicalist sense, life is prior to suffering, and life is more than this conversation about suffering and what to do about it. Procreating, consuming, growing, secreting, growing some more, always dying as newness is always born in each living moment - these are the experiences of living, not just suffering. And now life is thinking and writing or reading, not only suffering. Antinatalism isn’t just a tidy little syllogism categorized as ethics. It’s an act in the world, and an against life, which is procreative. Against suffering on paper, but inflicted upon all human life in action.

    Antinatalism, might simply be something psychological, a justification for suicidal tendancy applied on a universal scale.

    Mother Nature made use of suffering to fashion we species of ethical monkeys, only so that we could end the infliction of Her suffering on us and call it “good ethics.” Seems potentially delusional to have out smarted Mother Nature and her sufffering ways called “life.” With our “ethics” no less.

    Then it would stand to reason you are an anti-abortionist?AmadeusD

    That’s interesting.

    Antinatalism promotes no more babies because making a baby is the infliction of suffering on that baby. No one wants to inflict suffering on a baby, because it is just wrong. If that is the right way to live, and someone gets pregnant, the pregnant couple would have done wrong and inflicted suffering on another (or be in the act of inflicting suffering on another growing into such person).

    What can the antinatalist do with the new fetus? Can they abort it?

    If they can abort it, it must not be a person, because I would think the rule is that it is not ethical to kill another innocent person. That’s worse than inflicting suffering.

    But this is interesting. The antinataliat who doesn’t think a fetus is a person and who supports abortion would have to agree with the following: it is unethical to cause a sperm and an egg to form a fetus because that would be inflicting suffering on another person, but is it ok to kill the fetus after it is formed because a newly conceived fetus isn’t a person.

    Doesn’t an antinataliat have to be an anti-abortionist to lay out a consistent treatment of future people we do not want to inflict things upon?
  • schopenhauer1
    10.3k
    I might not only have to be an antinatalist, I might have to be an anti-hydrationist, because giving a thirsty person a glass of water, is like giving birth to a new person.Fire Ologist

    You still haven’t paid attention to preventative vs mitigative. One you’re already existing and now mitigation measures are appropriate. The other, you prevent the harm in the first place, as putting them in a situation in the first place which must be mitigated, is the ethical consideration in question. Hence, unique in moral choices.

    Bottom line to me, in a raw, physicalist sense, life is prior to suffering, and life is more than this conversation about suffering and what to do about it. Procreating, consuming, growing, secreting, growing some more, always dying as newness is always born in each living moment - these are the experiences of living, not just suffering. And now life is thinking and writing or reading, not only suffering. Antinatalism isn’t just a tidy little syllogism categorized as ethics. It’s an act in the world, and an against life, which is procreative. Against suffering on paper, but inflicted upon all human life in action.Fire Ologist

    In existential terms this is bad faith. You are assuming the role of arbiter of life. Rather this is simply something you chose to take on, it is not some robotic imperative. We are deliberative and can choose non life by choice, so prima facie, this appeal to the species or life is a false narrative.

    Mother Nature made use of suffering to fashion we species of ethical monkeys, only so that we could end the infliction of Her suffering on us and call it “good ethics.” Seems potentially delusional to have out smarted Mother Nature and her sufffering ways called “life.” With our “ethics” no less.Fire Ologist

    But ethical monkeys like us have ideas of antinatalism and choosing no suffering for others. You are anthropomorphizing nature. Nature doesn’t care whether the species dies or you or I fell off a cliff.
  • Fire Ologist
    381
    You still haven’t paid attention to preventative vs mitigative.schopenhauer1

    It get it, antinatalism is a preventative act in your mind. Analogies to mitigative acts don’t impress you. These distinctions don’t impress my view of antinatalism.

    You still don’t pay attention to much else I’ve written as well.

    Nature doesn’t care whether the species dies or you or I fell off a cliff.schopenhauer1

    Neither does nature care about any ethics at all, be it telling you to have 20 babies or 0. Neither does an unborn baby care what you inflict on it or not. Neither will anything care that there once were these ethical creatures who were so ethical they wouldn’t wantonly inflict their ethicalness on life anymore.

    I disagree that life is suffering. False premise.

    I disagree that procreating is an act upon a person - we don’t “inflict” anything when we participate willingly in the natural act of procreation. We aren’t acting for or against any particular human being when we procreate. The particular human being comes afterwards because life is prior to all of this ethical speak and life is prior to the harm of inflicting suffering by anyone or any process. Procreation is a choice to accept new life - not a choice to make nature do nature’s thing. Nature does the procreating - we accept it. We don’t inflict it (even in vitro).

    I disagree that it is always wrong to inflict suffering on another person. Just not a solid, clear, ethical basis to wipe out procreation. Inflicting suffering can be the most ethical thing to do.

    It is wrong to take what you are given and squander it, waste it, hoard it, and wrong to not share it, to not give it freely to others.

    Don’t have children if you think life is so terrible that no one should be forced to exist.

    Look down on people who do have children for their breaches of your ethics if you want.

    But Antinatalism is unethical. It misses the point of ethics entirely, which is good human life.
  • schopenhauer1
    10.3k
    Neither does nature care about any ethics at all, be it telling you to have 20 babies or 0. Neither does an unborn baby care what you inflict on it or not. Neither will anything care that there once were these ethical creatures who were so ethical they wouldn’t wantonly inflict their ethicalness on life anymore.Fire Ologist

    So again, for another time, the point is about the act of the parent, not the child. Do YOU (the potential parent) want to prevent suffering for another, if you can? It doesn't matter one wit, whether the child exists to KNOW this. It is about what YOU should do as the already existing person with the agency and means to do so.

    Now, the obvious response to this is something like what you are saying here:
    I disagree that procreating is an act upon a person - we don’t “inflict” anything when we participate willingly in the natural act of procreation. We aren’t acting for or against any particular human being when we procreate. The particular human being comes afterwards because life is prior to all of this ethical speak and life is prior to the harm of inflicting suffering by anyone or any process. Procreation is a choice to accept new life - not a choice to make nature do nature’s thing. Nature does the procreating - we accept it. We don’t inflict it (even in vitro).Fire Ologist

    But you will "inflict" something to "someone". It again, is a bad faith argument to understand what procreation is (leads to a new person), and then say that "because we don't know the exact person who will be born, that they don't exist yet, that the act has no affect/effect to someone". That is just patently false. Any future situation by definition, doesn't occur yet, that doesn't mean people don't have agency now that would lead to various known outcomes in the future. This is just rhetorically hollow argument, and a poor one that lands flat just on the face of it based on the nature of what procreation entails, what even the concept of "future" entails, and what human agency entails.

    But let's have a mind-journey here...
    Me: Once born, we know that people will die. Is this acceptable?
    Interlocutor: "Yes, death isn't bad in and of itself. It's the price we should pay for life".
    Me: Ok, some people think this is a harm, but let's grant that this is acceptable..
    Me: Once born, we are exposed to all kinds of illnesses and unfortunate circumstances including but not limited to:
    Accidents, addiction, aging, allergies, Alzheimer's disease, anxiety, arthritis, asthma, bankruptcy, bereavement, betrayal, birth defects, blindness, broken bones, bullying, burns, cancer, car accidents, chronic pain, chronic illnesses, cognitive decline, crime victimization, cystic fibrosis, depression, divorce, domestic violence, drowning, eating disorders, economic hardship, environmental disasters, epilepsy, eviction, falls, family conflict, financial instability, food poisoning, fraud, genetic disorders, grief, harassment, heart disease, homelessness, hunger, hurricanes, identity theft, imprisonment, infections, infertility, influenza, insomnia, job loss, kidnapping, loneliness, loss of a loved one, malaria, malnutrition, mental illness, migraines, natural disasters, neglect, neurological disorders, obesity, pandemics, Parkinson's disease, personal injury, pest infestations, physical abuse, poverty, pregnancy complications, PTSD, racial discrimination, respiratory infections, rheumatism, robbery, schizophrenia, school shootings, self-harm, separation, sexual abuse, sexual harassment, skin diseases, social isolation, stalking, starvation, stroke, substance abuse, suicidal thoughts, terminal illness, terrorism, theft, trauma, unemployment, violence, vision loss, war, water contamination, workplace accidents, xenophobia, yeast infections, zoonotic diseases, abandonment, abduction, abuse, acid attacks, acne, ADHD, aging parents, agricultural accidents, alcohol poisoning, amputation, anorexia, anxiety disorders, aphasia, appendicitis, asbestos exposure, asthma attacks, autoimmune diseases, avalanches, bed bugs, bedwetting, binge eating, bipolar disorder, blizzards, blood disorders, broken heart, bullying at work, burnout, cancer recurrence, carbon monoxide poisoning, carpal tunnel syndrome, cataracts, child labor, chronic fatigue syndrome, cluster headaches, colon cancer, communicable diseases, concussions, congenital heart defects, contagious diseases, COPD, Crohn's disease, cyberbullying, cystitis, debt, defamation, dental problems, dermatitis, developmental disorders, discrimination, dissociative disorders, drowning incidents, dyslexia, eclampsia, elder abuse, elephantiasis, emotional abuse, embezzlement, endometriosis, environmental pollution, eviction from home, explosive accidents, extortion, eye infections, famine, family estrangement, famine, farsightedness, fecal impaction, female genital mutilation, fibromyalgia, floods, flu pandemics, food allergies, foodborne illnesses, forced displacement, foreclosure, frostbite, gambling addiction, gas leaks, gastroenteritis, gender-based violence, genocide, glaucoma, global warming, gout, hangovers, hate crimes, head injuries, hearing loss, heatstroke, heavy metal poisoning, hepatitis, heroin addiction, high blood pressure, house fires, human trafficking, hurricanes, hyperthyroidism, hypothermia, hysterectomy complications, identity crises, illiteracy, incarceration, industrial accidents, influenza outbreaks, insect bites, interstitial cystitis, involuntary unemployment, irritable bowel syndrome, jaundice, jet lag, joint dislocations, judicial errors, kidney disease, landslides, language barriers, lead poisoning, leukemia, lice infestations, liver disease, lupus, Lyme disease, macular degeneration, malaria outbreaks, marital problems, measles, meningitis, meningococcal disease, mental breakdowns, migraines, mining accidents, miscarriage, mood disorders, multiple sclerosis, muscular dystrophy, narcissistic abuse, narcolepsy, neck injuries, neurodegenerative diseases, neurofibromatosis, neuropathy, obesity-related complications, occupational hazards, obsessive-compulsive disorder, organ failure, osteoarthritis, osteoporosis, pancreatitis, panic attacks, paraplegia, parental neglect, parasitic infections, peptic ulcers, peripheral artery disease, pesticide exposure, phobias, physical deformities, pink eye, plagiarism, pneumonia, police brutality, post-natal depression, premature birth, prostate cancer, psoriasis, psychosis, radiation exposure, radon poisoning, rape, respiratory failure, restless leg syndrome, retaliation, rickets, road rage incidents, roof collapses, salmonella poisoning, sanctions, SARS, scabies, schizophrenia episodes, scoliosis, seasonal affective disorder, seizures, septicemia, severe weather conditions, sexual exploitation, shingles, sinkholes, skin cancer, sleep apnea, sleep deprivation, smoking addiction, social anxiety disorder, soil erosion, speech disorders, spinal cord injuries, stalking incidents, stillbirth, stomach cancer, stress, stroke recovery, sudden infant death syndrome, suicidal ideation, surgical complications, swine flu, tax evasion charges, terrorism threats, thyroid cancer, ticks, tornadoes, toxic relationships, traffic congestion, trafficking in persons, transphobia, traumatic brain injuries, trench foot, tuberculosis, typhoid fever, ulcers, underemployment, unemployment, urban decay, uterine cancer, vandalism, vehicle theft, venomous bites, viral infections, volcanic eruptions, war injuries, wasting syndrome, water scarcity, whiplash, whooping cough, wildfires, withdrawal symptoms, workplace harassment, workplace stress, worms, wrist injuries, wrongful accusations, wrongful death, yellow fever, zoonotic infections, noise pollution, office politics, unreasonable bosses, misunderstandings, broken friendships, peer pressure, unrealistic societal expectations, cultural alienation, discrimination, traffic accidents, loneliness in a crowd, missed opportunities, creative blocks, existential crises, dealing with bureaucracy, feeling unfulfilled, high cost of living, technology failures, power outages, scarcity of resources, environmental degradation, emotional exhaustion, systemic corruption, loss of cultural heritage, internet addiction, digital divide, cyberattacks, data breaches, peer rejection, public speaking anxiety, negative body image, climate anxiety, overwork, burnout, imposter syndrome, midlife crisis, family obligations, cultural expectations, aging infrastructure, housing shortages, inaccessible healthcare, poor work-life balance, stigma of illness, fear of aging, generational conflict, substance dependencies, cyberstalking, deforestation, plastic pollution, extreme weather, droughts, heatwaves, rising sea levels, ocean acidification, social media harassment, self-esteem issues, fear of failure, infertility treatments, invasive medical procedures, chronic stress, overpopulation, resource wars, political instability, societal unrest, ideological conflicts, legal battles, copyright infringement, loss of reputation, deteriorating mental health, peer pressure, unfulfilled potential, financial scams, ergonomic injuries, urban sprawl, toxic friendships, loss of tradition, dealing with dementia in family members, balancing career and family, navigating personal identity, confronting mortality, technological obsolescence, scarcity of natural resources, global economic shifts, cultural misappropriation, fear of public places, imposter syndrome, financial market crashes, fear of flying, language extinction, misinformation, corporate espionage, loss of biodiversity, noise pollution, space junk, invasive species, soil depletion, and burnout.

    Now, you may say that this is contingent and we don't know what people will really endure. And I would say, that i correct, and why I would call them "contingent/circumstantial harms". And thus, the beginning of this particular argument was showing that the Veil of Ignorance applies here, because (contra other posters here like TClark), we do not have even imperfect knowledge of what types and degrees of harms people will endure once born.

    Then you will tell me, "No we do know to some degree the amount of harms, based on what we've experienced". But then, I would say the crux of the issue regarding dignity pops up. That is to say, what are you willing to allow another person to be exposed to in your pursuit of X? In this case, it looks like your X is an impersonal idea of "Life" or "Mother Nature". None of these things are entities that deserve the category of ethical consideration. The only people that matter are the people that might be born who will be affected. Those are the loci of actual pain and suffering.

    Then you can tell me that the "goods" outweigh the pain.. That "Mother Nature" and "Life" deserves such-and-such. Besides being an impersonal concept, and thus a category error for ethical consideration that deserves absolutely nothing, being "deserves" doesn't even apply to such impersonal concepts.. We have to understand what it is about suffering that makes it weighted as more important to prevent than promoting-good/happiness. This goes to the notion of aggressive paternalism.

    Just as you say here:
    Don’t have children if you think life is so terrible that no one should be forced to exist.

    Look down on people who do have children for their breaches of your ethics if you want.
    Fire Ologist

    Acknowledging that people have a right to their own estimation of things...
    It is not just/fair to estimate for others, a whole lifetime of what various harms and suffering is acceptable. In fact, not considering suffering when they are not in a scenario where they need help (already exist), is purely using someone, because precisely because that person doesn't exist yet, any move to create that person, would always be one whereby one is inflicting that suffering upon them when it did not need to be inflicted so. Thus it is always the case that this is using someone as a means.

    Adding Benatar's asymmetry (reformulated and tweaked a bit), this is no debate.
    Preventing procreation prevents suffering and happiness. Preventing suffering is the obligation, preventing happiness matters not. And no "thing" will cry over spilt milk, not Mother Nature, not Life.

    It is aggressively paternalistic to assume that your unhappiness for other people not experiencing X positive value, is the reason why people should thus suffer. And as I explained, certainly, Mother Nature and Life, abstract and reified ideas, have no say in the matter.
  • Fire Ologist
    381
    So again, for another time, the point is about the act of the parent, not the child. Do YOU (the potential parent) want to prevent suffering for another, if you can?schopenhauer1

    Wrong. YOU are still talking about the child too. You should be saying something like this: Do you, the potential parent want to be a person who inflicts suffering, do you want to walk around being an unethical person who inflicts suffering, or do you want to be be ethical? Takes the child out of the equation. If it’s not about the child then it is not about preventing suffering “for another”.

    You can’t make your arguments without committing the same error you accuse me of making.

    It’s not an error. You need two existing subjects before you can judge the actions of one upon the other as ethical or not. The best you can say is the present world would be a better place if it was filled with people who acted according to principle and and acted according to ethical principle. If one of those principles happens to mean all people will cease to exist, the world is still better today because all of us principled antinatalists inhabit it.

    That’s your argument. But you said “for another” anyway.

    This is just rhetorically hollow argumenschopenhauer1

    Almost as hollow as thinking we humans, the sole source of ethics, came to be this way by a natural process that was unethical all along.

    Don’t you think there is as long a list of great things that happen to people as your laundry list of dirty laundry? You need the clean clothes first before you can get stuck with the dirty laundry. You need life first, apart from suffering, free from suffering, to later suffer anything.

    what are you willing to allow another person to be exposed to in your pursuit of X? "schopenhauer1

    What are you willing to take away from another person in your pursuit of your ethical ideals? Take away parenting? Take away loving your children. Take away pride in how those children endure and learn from suffering and are charitable with their sacrifices?

    Life does include suffering. Everyday to some extent, for everyone and every creature who lives, there is suffering.

    I also don’t want to inflict any more suffering on anyone to make matters worse for them.

    But, what is the point of being ethical when being ethical means there will no longer be beings being ethical?

    Antinatalism is anti-ethics.

    Which is why earlier I said antinatalism puts ethical principals above the people who are being ethical, and does so having the effect of there no longer being any people. So what is the point of being ethical when ethics itself is being used to destroy is, to harm the species, to inflict upon the world a world without any ethics (without people)?

    It’s all backwards and confused. Like murdering someone for their own good.
  • Leontiskos
    1.7k
    You claim that the antinatalist is secretly religious...schopenhauer1

    No, I am saying that antinatalism is non-naturalistic. I wrote out responses to all of your claims, but then I shelved that post because it was so long. Instead I focused on the simple argument () and the foundational Gnosticism ().

    Regarding the simple argument, I will ask you a simple question: Is it true that we should not procreate in a world where everyone receives one pinprick of pain followed by 80 years of pure happiness? Yes or no? Presumably some have ambitions of achieving such a world someday.

    Here is a easier way to envision the foundational Gnosticism problem. The evolutionary naturalist says:

    1. Reasoning, including moral reasoning, is a result of evolutionary adaptation
    2. Evolutionary adaptation is ordered to survival
    3. Therefore, moral reasoning is ordered to survival
    4. The argument which says we should cease procreating would lead to extinction
    5. Therefore this argument is unsound; contrary to evolutionary adaptation ordered to survival

    This argument is a microcosm of the anti-Gnosticism argument, for the Gnostic must reject naturalistic premises akin to (1). For the naturalist, antinatalism is by definition irrational, as it is directly contrary to nature. Your arguments are all dubious, but one of the fundamental reasons they are all dubious is because you are essentially importing knowledge from a different "god," a god that is foreign to our nature, culture, religions, etc. In denying moral naturalism you must necessarily be appealing to some form of supernaturalism. Think of it in terms of the microcosm: if your ethic is directly contrary to evolutionary survival, then it must be coming from something above and beyond evolutionary survival.
  • schopenhauer1
    10.3k
    Wrong. YOU are still talking about the child too. You should be saying something like this: Do you, the potential parent want to be a person who inflicts suffering, do you want to walk around being an unethical person who inflicts suffering, or do you want to be be ethical? Takes the child out of the equation. If it’s not about the child then it is not about preventing suffering “for another”.Fire Ologist

    Don't look now, but you are making my arguments...
    Yeah reformulate it to take the child out of the equation. It's about the parent.
    However, I am not opposed to talking in "potentials" or "future children". What I AM opposed to is talking AS IF the future child will feel "deprived" of not existing. So I think it's fine to say, "You can prevent future suffering", because it is the NOT SUFFERING that matters. That someone DOES NOT exist to suffer is the ethically good outcome. That someone DOES NOT EXIST TO BE HAPPY matters not one wit..

    Almost as hollow as thinking we humans, the sole source of ethics, came to be this way by a natural process that was unethical all along.Fire Ologist

    Again, category error to input ANY ethical thing to nature. Humans are too plastic for this kind of thinking. Did nature intend us to have computers? No, nature intended nothing. Nature has nothing to say on nothing. Once you have degrees of freedom of deliberation, it is up to us to figure shit out and not go bad faith and say, "What did nature want?". As nature a) isn't something that can confer morality and b) nature doesn't tell us about morality. And any use of "appeal to nature" can be to justify anything because we are brutes and we do ethical things as well.. it is up to us to deliberate on what to sus out as the correct view and action. There is nothing to fall back on. Even if we did, it would be your interpretation of it. Or at best using descriptive ethics to justify normative ethics.

    Don’t you think there is as long a list of great things that happen to people as your laundry list of dirty laundry? You need the clean clothes first before you can get stuck with the dirty laundry. You need life first, apart from suffering, free from suffering, to later suffer anything.Fire Ologist

    And that is my point, when not under mitigating circumstances, but preventative choices, mitigating circumstances don't matter. If you allow/inflict/not prevent suffering even with the intent on someone enjoying the good, but knowing you are allowing the bad, that doesn't mean that there wasn't a violation. Causing suffering that was not necessary to cause because you have an agenda is the height of paternalism and means-using.

    What are you willing to take away from another person in your pursuit of your ethical ideals? Take away parenting? Take away loving your children. Take away pride in how those children endure and learn from suffering and are charitable with their sacrifices?Fire Ologist

    None of these things are ethical. In Kantian terms, they are simply "inclinations" or ends that people might want to pursue. They aren't moral onto themselves like suffering prevention is.

    I also don’t want to inflict any more suffering on anyone to make matters worse for them.Fire Ologist

    That's good. But you are in the realm of mitigation now, since "not make worse", not prevention.

    But, what is the point of being ethical when being ethical means there will no longer be beings being ethical?Fire Ologist

    Because ethics is around because there are humans around, humans aren't around to keep ethics going, for the nth time. In fact, it would be immoral to keep people around to keep some ethical framework going, as that is exactly the reason it is unethical in the first place- it is discounting the causation of someone else's lifetime of suffering for the promotion of some X ethical project that you conceive of!

    It’s all backwards and confused. Like murdering someone for their own good.Fire Ologist

    Actually, quite the opposite. Death comes from being born in the first place. All the sufferings of life, including being murdered come from one's choice to overlook suffering for one's project. And since no one is exists to feel the "deprivation" of good, go back to the spilt milk argument.
    Regarding the simple argument, I will ask you a simple question: Is it true that we should not procreate in a world where everyone receives one pinprick of pain followed by 80 years of pure happiness? Yes or no? Presumably some have ambitions of achieving such a world someday.Leontiskos

    And I gave you my answer....

    Also, about "someday", I wouldn't make people in the meantime become grist for this goal.

    Reasoning, including moral reasoning, is a result of evolutionary adaptationLeontiskos

    This is not fact. Moral reasoning isn't necessarily evolved for from evolutionary reasons, but could be a secondary byproduct of other reasons. I do know that whole fields like evolutionary psychology exist, but it's often contested certain methodologies being it's near impossible to know the original environment conditions of our ancestors, and what particular traits were evolved for versus simply having large brains with a degree of plasticity. For examples, presumably making computers wasn't "evolved for", but perhaps basic numeracy was. It would be bad faith arguing to presume you are going to get from basic structures to complex ones without it being a "just so" story. And spare me any articles, I understand that these theories exist.. Doesn't mean they are hard facts.

    Evolutionary adaptation is ordered to survivalLeontiskos

    Evolutionary adaptation isn't "ordered" per se, but rather, is a shotgun approach of genetic variation against environmental factors. It prunes away variations not fit for the environment reproduction. The order is simply our way of organizing the "real time" of this happening in nature. The "order" is not like some "logos", just how we humans are understanding the variations that cause change in species fitting into environmental niches.

    Therefore, moral reasoning is ordered to survivalLeontiskos

    Again, not necessarily the case. Moral reasoning might be even a more modern thing really. It could have been traditional understanding based on tribal traditions. I wouldn't call that reasoning as an appeal to tradition. For example, let us say there is a culture with a history of some heinous act, but the heinous act allowed them to survive. I wouldn't say this is "moral reasoning". There are blood feuds and slavery and honor killings, and all such things that have kept people in line. This doesn't mean it's moral. There may be stuff that helped to survive that is actually immoral.

    The argument which says we should cease procreating would lead to extinction
    Therefore this argument is unsound; contrary to evolutionary adaptation ordered to survival
    Leontiskos

    This is the same problem as Fire Ologist. Evolution has nothing to say about morality, and anything we glean from it is from post-facto reasoning, not because it is inbuilt. It is cultural, personal, existential, but not necessarily instinctual. There can arguments that basic empathy and fairness are seen in small children, but this is so general as to not be an actual ethical basis. I can say it is compassionate and fair to not procreate, and it just took being an adult with a certain amount of reasoning abilities to understand this, using my "evolutionary" traits of compassion and fairness.

    This argument is a microcosm of the anti-Gnosticism argument, for the Gnostic must reject naturalistic premises akin to (1). For the naturalist, antinatalism is by definition irrational, as it is directly contrary to nature. Your arguments are all dubious, but one of the fundamental reasons they are all dubious is because you are essentially importing knowledge from a different "god," a god that is foreign to our nature, culture, religions, etc. In denying moral naturalism you must necessarily be appealing to some form of supernaturalism. Think of it in terms of the microcosm: if your ethic is directly contrary to evolutionary survival, then it must be coming from something above and beyond evolutionary survival.Leontiskos

    Again, as far as I am willing to entertain Gnosticism, it is as a metaphor that existence has some inherent suffering aspects to it merely by being something with becoming and is not fully being. I can go into it more, but it would be Schopenhauer's idea of boredom being a sort of proof of the negative nature of happiness, as a repose rather than something that actually satiates. It is the treadmill rather than the peace of fully being.. Schopenhauer might say that "higher realm" is some sort of non-willing peacefulness of nirvana. I don't necessarily believe that is possible. @Wayfarer has called this belief "nihilistic" because there is no soteriology whereby one can escape the cycle of suffering.

    Let me bring you through a series of questions:
    What are you intending a new person born to get out of life? Why must there be someone versus no one that isn't selfish from the start (remember, no person exists prior to you to care)?

    Once born, let's say the thing you envision is some sort of happiness conceived of in whatever way (religious, social, personal, hedonistic, virtue, etc.). What if they fail on most of those ways?

    If your answer is that "They just have to figure out a better way".. how is this not paternalistically assuming that people "must do" this project that you had for them? It's forcing a set of (de facto) choices that these new people will have to pursue. Once "in the game", there is no way out of having to make these choices, and sometimes they lead to no good outcome for the player. It is then quite gaslighting to blame the player for not following the plan well enough the way you want it, or to say that it didn't work out in their favor, or to say if they "just did this or they would be better". I can't emphasize enough the kind of hubristic paternalism necessary in deciding for another that this project is what needs to be played out be a person that is someone that is not you simply because you have the power to create that person.
  • Fire Ologist
    381
    In denying moral naturalism you must necessarily be appealing to some form of supernaturalism. Think of it in terms of the microcosm: if your ethic is directly contrary to evolutionary survival, then it must be coming from something above and beyond evolutionary survival.Leontiskos

    Interesting point.

    An attack from a different flank.

    I’ve been pointing out that if the ethic was indeed a product of nature, it would be nature developing an ethic that led to the demise of all ethic agents and so the demise of ethics itself; the ethic that was naturally born would then have to turn and destroy itself to be an antinatalist ethic. I’ve been trying to show it is internally self-defeating - living things using their lives to end living things that use their lives ethically.

    But destroying itself is not what life naturally does. It’s just the opposite. Living things may devour their surroundings and destroy things, but life does so to live and to endure destructive forces from outside nature.

    This all just further supports your argument that antinatalism is not natural.
  • RogueAI
    2.6k
    This all just further supports your argument that antinatalism is not natural.Fire Ologist

    I don't see this as a knock on his position. Birth control isn't natural, but it's not immoral.
  • Fire Ologist
    381
    That someone DOES NOT exist to suffer is the ethically good outcome.schopenhauer1

    This is two parts: 1. That SOMEONE does not exist. And 2. “Exist to suffer”.

    You are talking about the child as if actual, not potential. You did a much worse job of making this about the parent. The ethically good outcome has to be about the parent, the ethical person acting ethically. Not someone else. Be they “not exist” as a potential child or “exist to suffer” as an actual child.

    Again, category error to input ANY ethical thing to nature. Humans are too plastic for this kind of thinking. Did nature intend us to have computers? No, nature intended nothing. Nature has nothing to say on nothing. Once you have degrees of freedom of deliberation, it is up to us to figure shit out and not go bad faith and say, "What did nature want?". As nature a) isn't something that can confer morality and b) nature doesn't tell us about morality. And any use of "appeal to nature" can be to justify anything because we are brutes and we do ethical things as well.. it is up to us to deliberate on what to sus out as the correct view and action. There is nothing to fall back on. Even if we did, it would be your interpretation of it. Or at best using descriptive ethics to justify normative ethics.schopenhauer1

    Don’t really need to reply here. I’m not talking about nature like it’s intentional. It’s causal. Mother Nature is a metaphor for causality, or natural necessity. Like a biological function. Like procreation. Like, in the case of humans, ethics. Ethics came from humans and humans came from natural processes so ethics sits directly in nature in us humans. Antinatalism would be nature’s human ethics that requires by natural necessity humans unnaturally stop procreation, which ends the ethics that sits only in humans which formerly sat in nature. Total mess.

    They aren't moral onto themselves like suffering prevention is.schopenhauer1

    Suffering prevention. Is this the highest ethic, the only ethic, a foundational ethic to all that are built on it? Or just another ethic where someone might hold some other ethic higher while keeping suffering prevention close, just not central?

    I think you have to say it is up there pretty close to your highest ethic. All other ethics might add some suffering to the world.

    Antinatalism sort of is a one size solution fits all human immorality solution.

    humans aren't around to keep ethics goingschopenhauer1

    What a phrase.

    And I know you don’t care about this but it means ethics is as meaningless as your suffering, your life, and your precious preventative sentiments. Why be ethical? It’s a different question, but antinatalism does not promote a good sense of meaning and purpose behind being ethical. It rids the world of the life out in place that would do the preventing of procreation.

    What are you intending a new person born to get out of life?schopenhauer1

    Who knows? No one can influence what a person gets for themselves out of this life. That’s up to them to get out of it what they can. To the ones who are born to us we can only give them things out of life - it is up to them and their intentions to take these and get things out of life. All we can intend is the same thing we can physically provide - an opportunity. It’s called procreation.

    how is this not paternalistically assumingschopenhauer1

    There’s nothing paternalistic by banning all babies? It is an ironic use of the term “paternal” but “thou shalt prevent suffering and never have children.” Just as wide open to derision for “paternalism”.

    You never responded to this:

    Antinatalism promotes no more babies because making a baby is the infliction of suffering on that baby. No one wants to inflict suffering on a baby, because it is just wrong. If that is the right way to live, and someone gets pregnant, the pregnant couple would have done wrong and inflicted suffering on another (or be in the act of inflicting suffering on another growing into such person).

    What can the antinatalist do with the new fetus? Can they abort it?

    If they can abort it, it must not be a person, because I would think the rule is that it is not ethical to kill another innocent person. That’s worse than inflicting suffering.

    But this is interesting. The antinataliat who doesn’t think a fetus is a person and who supports abortion would have to agree with the following: it is unethical to cause a sperm and an egg to form a fetus because that would be inflicting suffering on another person, but is it ok to kill the fetus after it is formed because a newly conceived fetus isn’t a person.

    Doesn’t an antinataliat have to be an anti-abortionist to lay out a consistent treatment of future people we do not want to inflict things upon?
    Fire Ologist

    Well?
  • Fire Ologist
    381
    I don't see this as a knock on his position. Birth control isn't natural, but it's not immoral.RogueAI

    It’s a more clear blow than my same point about it being unnatural. It just means you can’t be a naturalist if you want to be an antinatalist. You couldn’t have drawn the ethic from nature. Some other influence, like a god as an easy example whispered “no more babies lest you cause all their suffering.” If the antinatalist doesnt want to have a deity, fine then - but where did they get the idea of a universal morality that wipes out procreation was a good?

    I still say it makes no sense to prevent suffering in another by eliminating the other (through never making the other). You never have the prevention if you never have the other to prevent from something.
  • schopenhauer1
    10.3k
    This is two parts: 1. That SOMEONE does not exist. And 2. “Exist to suffer”.

    You are talking about the child as if actual, not potential. You did a much worse job of making this about the parent. The ethically good outcome has to be about the parent, the ethical person acting ethically. Not someone else. Be they “not exist” as a potential child or “exist to suffer” as an actual child.
    Fire Ologist

    No. Again, it IS from the perspective of the parent. You CAN project onto the future and see hypotheticals. Someone DID NOT SUFFER (that could have). There is no actual "someone", but from the perspective of the parent, this prevention was what was the ethical decision, and being a negative ethic, it is good that it DID NOT occur. And again, the ethic, being a negative ethic, was about NOT allowing the suffering. The asymmetry is when you project to the other way, "Happiness did NOT occur". This is not a violation; it's not obligatory to promote happiness, only prevent the situation where an ACTUAL someone would suffer. The ACTUAL someone did not suffer. Violation did not take place.

    As an aside, you can certainly disagree with the formulation and reformulate it. That is fine. The nature of the ethic doesn't change, just the language used for hypothetical scenarios. But oddly enough, however you formulate the language, we actually agree it is from the perspective of the person making the decision.

    Don’t really need to reply here. I’m not talking about nature like it’s intentional. It’s causal. Mother Nature is a metaphor for causality, or natural necessity. Like a biological function. Like procreation. Like, in the case of humans, ethics. Ethics came from humans and humans came from natural processes so ethics sits directly in nature in us humans. Antinatalism would be nature’s human ethics that requires by natural necessity humans unnaturally stop procreation, which ends the ethics that sits only in humans which formerly sat in nature. Total mess.Fire Ologist

    Natural necessity? This is all bad faith arguing. You are arguing as if humans cannot deliberate for themselves, or that there are directives from nature that force us to decide a certain way. Calling your argument by another name "causality/natural necessity", doesn't change the incoherent nature of the argument, thus everything imputed on your idea of "Mother Nature" remains for this new term, that also matters not to moral decisions. There is no logos, no natural necessity to fall back on.

    Suffering prevention. Is this the highest ethic, the only ethic, a foundational ethic to all that are built on it? Or just another ethic where someone might hold some other ethic higher while keeping suffering prevention close, just not central?

    I think you have to say it is up there pretty close to your highest ethic. All other ethics might add some suffering to the world.

    Antinatalism sort of is a one size solution fits all human immorality solution.
    Fire Ologist

    Indeed negative ethics, like not violating rights, or making people suffer unduly, would be pretty high up there, yes. The basis could be several things, but it is based on a deontology of not using people in this case. YOUR positive projects shouldn't be at the behest of SOMEONE ELSE'S negative violation. The right not to be used/rights violated/negative ethics violated is above someone else's notion of what is best to occur in the world.

    What a phrase.

    And I know you don’t care about this but it means ethics is as meaningless as your suffering, your life, and your precious preventative sentiments. Why be ethical? It’s a different question, but antinatalism does not promote a good sense of meaning and purpose behind being ethical. It rids the world of the life out in place that would do the preventing of procreation.
    Fire Ologist

    No, I am not sure why this phrase or concept is so puzzling to you. Let's say instead of ethics, it was "Make plastic". Would it make sense that humans need to be around so plastic exists? That makes no sense. Nothing "needs to be around", including ethics. It's a byproduct of what humans face when making decisions. If there is no potential for ethical considerations (like procreation, murder, theft, etc.) to exist, then there are no need for ethics. It's not much more complicated than that.

    Who knows? No one can influence what a person gets for themselves out of this life. That’s up to them to get out of it what they can. To the ones who are born to us we can only give them things out of life - it is up to them and their intentions to take these and get things out of life. All we can intend is the same thing we can physically provide - an opportunity. It’s called procreation.Fire Ologist

    Right..so this speaks to exactly my point about the intent versus reality, Veil of Ignorance, and creating projects for someone else. I know you don't see anything wrong here, and that is part of our miscommunication/disagreement. I at least want you to understand where the disagreement is, even if you disagree. I don't know if you see it here with your answers to these questions..

    There’s nothing paternalistic by banning all babies? It is an ironic use of the term “paternal” but “thou shalt prevent suffering and never have children.” Just as wide open to derision for “paternalism”.Fire Ologist

    No, that would mean all ethics that say "Don't do X" is paternalistic. That would be absurd as any action that is an "ought" would be this, and we would be abusing language. "Don't murder" is not paternalistic. Rather, unlike the antinatalist ethic, which PREVENTS a paternalistic scenario from happening, procreation WILL cause another person to be affected, to be born for X reason/project. THAT is paternalistic to think another person should be allowed to suffer because you have a notion of X. No one is affected by anything by not procreating/antinatalist argument. So by definition and applying it to the outcome, antinatalism cannot be paternalistic like procreation would indeed be. Also, being that almost no antinatalist philosophies entail being "forced" to not have children, but rather provide an ethical reason not to, there is no problem here either. The person can freely choose what they think is right.

    But this is interesting. The antinataliat who doesn’t think a fetus is a person and who supports abortion would have to agree with the following: it is unethical to cause a sperm and an egg to form a fetus because that would be inflicting suffering on another person, but is it ok to kill the fetus after it is formed because a newly conceived fetus isn’t a person.

    Doesn’t an antinataliat have to be an anti-abortionist to lay out a consistent treatment of future people we do not want to inflict things upon?
    Fire Ologist

    I didn't answer because it's a non-sequitur. This isn't a debate about abortion. An antinatalist is not entailed to believe anything regarding abortion, but certainly, one who believes that the fetus (qualified perhaps by a certain time period of gestation), is not a person yet, would think that abortion is permissible.
  • Fire Ologist
    381
    I didn't answer because it's a non-sequitur. This isn't a debate about abortion. An antinatalist is not entailed to believe anything regarding abortion, but certainly, one who believes that the fetus (qualified perhaps by a certain time period of gestation), is not a person yet, would think that abortion is permissible.schopenhauer1

    So an antinatalist can say it is impermissible to procreate, to create a new fetus, in order to prevent suffering, but it is ok to kill a human fetus.

    Got it.
  • SpaceDweller
    514
    I didn't read entire thread and don't know if this was discussed already but isn't anti-natalism perfect solution to overpopulation?

    It's not new that overpopulation is a problem, and killing off billions of people is unacceptable, but Anti-natalism would solve that problem in only 50 years without anyone suffering.
  • schopenhauer1
    10.3k
    I didn't read entire thread and don't know if this was discussed already but isn't anti-natalism perfect solution to overpopulation?

    It's not new that overpopulation is a problem, and killing off billions of people is unacceptable, but Anti-natalism would solve that problem in only 50 years without anyone suffering.
    SpaceDweller

    Antinatalism can take many forms, but most antinatalism is not about preventing various environmental or population outcomes but due to not wanting to allow suffering for a future person. Indeed, a result would be a reduced people born, which would in fact reduce the population, but that is generally not the position of most philanthropic or misanthropic antinatalists. Once the population levels off to a "correct" level, the environmental/population "antinatalist" would no longer be so, and thus was a contingent natalist, really.

    Interestingly, in the US, countries in Western Europe, Japan, Korea, Russia, and even China (even AFTER the one child policy restrictions) the population has gone to below replacement levels which makes economists and social conservatives uneasy. They need little units of labor to keep the pyramid scheme going, and this "lower than replacement level" gives them pause for alarm. Here are some recent stories on this:

    https://www.cnn.com/2024/03/20/health/global-fertility-rates-lancet-study/index.html
    https://www.wsj.com/us-news/america-birth-rate-decline-a111d21b
    https://www.economist.com/leaders/2023/06/01/global-fertility-has-collapsed-with-profound-economic-consequences?utm_content=section_content&gad_source=1&gclid=CjwKCAjw7NmzBhBLEiwAxrHQ-Z7JGafAcUZiAufKxKdpfX4yzbiPVxb0FOEcHE1d1-oiBo7YEc6WLRoC7cwQAvD_BwE&gclsrc=aw.ds
    https://www.vox.com/23971366/declining-birth-rate-fertility-babies-children

    https://time.com/6836949/birth-rates-south-korea-japan-decline/
    https://www.ft.com/content/008a1341-1882-4b98-83d4-0d7dc08a4134
    https://www.nbcnews.com/news/world/china-fertility-rate-record-low-rcna100353

    Now I'd love to think this worldwide decline in fertility rate is because everyone realizes that bringing more people into the world means bringing more suffering into the world, and they want to prevent this in a noble ethical way, as good antinatalists.. But that is not the reason. The reason of course is mainly due to the fact that the more educated a population, and higher the standard of living, the harder it is to raise a child. People also end up living more individualistic lifestyles that tend to skew child-free over family-type living. So it is more about lifestyle choice, the cost of raising a child, and a more individualistic, less traditional view of goals in life.

    Of course in poorer, less educated (for women especially), and more traditional societies like the Middle East, parts of South America, Africa, and Asia, the fertility rate is still fairly high and so in those areas you will continue to see growth, and the overall growth will continue being that there's simply more people on the planet in general, but it will slow.

    Either way, no, population decline is generally not the goal of antinatalists. Since population reduction is context dependent (as presumably it would be permissible to have children again once sustainable levels are reached) we maybe can call it "context-dependent antinatalism". I guess in theory as well, "misanthropic" antinatalism falls under this (rather than philanthropic ... or "to prevent suffering").. That is to say, people think that humans are ruining the environment, other animals, habitats, and the current living conditions and thus need to stop breeding until such time as these recover. Usually these people are simply for fewer children, not absolutely no children.
bold
italic
underline
strike
code
quote
ulist
image
url
mention
reveal
youtube
tweet
Add a Comment

Welcome to The Philosophy Forum!

Get involved in philosophical discussions about knowledge, truth, language, consciousness, science, politics, religion, logic and mathematics, art, history, and lots more. No ads, no clutter, and very little agreement — just fascinating conversations.